Tải bản đầy đủ (.pdf) (10 trang)

Chứng minh một số bất đẳng thức bằng phương pháp so sánh giá trị của đồ thị lồi, lõm tại các điểm cực biên

Bạn đang xem bản rút gọn của tài liệu. Xem và tải ngay bản đầy đủ của tài liệu tại đây (411.25 KB, 10 trang )

Hội thảo Khoa học, Sầm Sơn 28-28/09/2019

C HỨNG MINH MỘT SỐ BẤT ĐẲNG THỨC BẰNG
PHƯƠNG PHÁP SO SÁNH GIÁ TRỊ CỦA ĐỒ THI LỒI ,
LÕM TẠI CÁC ĐIỂM CỰC BIÊN
Nguyễn Văn Nhiệm
Trường THPT Chuyên Lam Sơn, Thanh Hóa

Tóm tắt nội dung
Một trong những con đường hình thành nhận thức đó là “Từ trực quan sinh động
đến tư duy trừu tượng”, từ các ví dụ cụ thể đến khái niệm tổng quát. Trong bài viết này
trình bày chứng minh một số bất đẳng thức hay và khó, trong các kì thi học sinh giỏi
bằng cách nhìn vào điểm mút của đồ thi lồi, lõm. Một hình ảnh trực quan sinh động mà
mọi học sinh đều dễ dàng nhận thấy bằng hình học. Đặc biệt đồ thị của đoạn thẳng có
thể xem là đồ thị lồi, cũng có thể xem là đồ thị lõm, sẽ được áp dụng vào chứng minh
bất đẳng thức.

1

Lí thuyết

Định lý 1.1. Nếu hàm số f ( x ) = ax + b thoả mãn f (α) ≥ 0 và f ( β) ≥ 0, (α < β) thì
f ( x ) ≥ 0, ∀ x ∈ [α; β]. Chứng minh. Đồ thị của hàm số f ( x ) = ax + b là một đường
thẳng, nên theo tính chất của đoạn thẳng: ”Nếu hai đầu mút của đoạn thẳng là hai
điểm A(α, f (α)), B( β, f ( β)) ở phía trên trục hồnh thì đoạn thẳng đó nằm phía trên trục
hồnh”.

Định lý 1.2. Nếu f ( x ) = ax + b, thì min { f (α), f ( β)} ≤ f ( x ) ≤ max { f (α), f ( β)} , ∀ x ∈
[α, β].
Mở rộng:
Định lý 1.3. i) Nếu f ( x ) là hàm lồi (đồ thị quay bề lõm lên phía trên) trên [α, β] thì


f ( x ) ≤ max { f (α), f ( β)} , ∀ x ∈ [α, β]. ii) Nếu f ( x ) là hàm lõm (đồ thị quay bề lõm
xuống phía dưới) trên [α, β] thì f ( x ) ≥ min { f (α), f ( β)} , ∀ x ∈ [α, β].

1


Hội thảo Khoa học, Sầm Sơn 28-28/09/2019

2

Áp dụng

Bài toán 2.1 (IMO -84). Cho x, y, z là các số thực không âm sao cho x + y + z = 1. Chứng
7
. Dấu đẳng thức xảy ra khi nào?
minh rằng 0 ≤ xy + yz + zx − 2xyz ≤
27
Cách giải. vì x, y, z ∈ [0; 1] ⇒ 2xyz ≤ xy + yz ⇒ xy + yz + zx − 2xyz ≥ xz ≥ 0. Dấu
đẳng thức xảy ra khi hai số bằng 0 và một số bằng 1.
7
7
Đặt t = yz ⇒ xy + yz + zx − 2xyz −
= (1 − 2x )t + x (1 − x ) −
= f ( t ), 0 ≤ t ≤
27
27
2
(1 − x )
.
4

7
1
7
1
(1 − x )2
Ta có f (0) = x (1 − x ) −


= −
< 0, và f
=
27
4
27
108
4
−54x3 + 27x2 + 1
( x − 1/3)2 (6x + 1)
= −
≤ 0. Dấu đẳng thức xảy ra khi x = y =
108
12
1
z= .
3
Bài toán 2.2. Cho a, b, c ∈ [0; 1], a + b + c = 2. Chứng minh rằng ab + bc + ca ≥ 2abc +
2

20
.

27

2
(2 − a )
Cách giải. Giả sử a ≥ b ≥ c ⇒ 1 ≥ a ≥ . Đặt t = bc ⇒ 0 ≤ t ≤
. Bđt
3
4
20
20
⇔ f (t) = (1 − 2a)t + a(2 − a) −
≥ 0. Ta có f (0) = − a2 + 2a −
> 0 và f (t) =
27
27
2
−54( a − 32 ) ( a − 76 )
−54a3 + 135a2 − 108a + 28
=
≥ 0.
108
108
Bài toán 2.3. Cho a, b, c, d ∈ [0; 1]. Chứng minh rằng a + b + c + d − abcd ≤ 3.
Cách giải. Đặt f ( a, b, c, d) = a + b + c + d − abcd. Ta có
f ( a, b, c, d) = a + b + c + d − abcd ≤ max { f (0, b, c, d), f (1, b, c, d)}
.
f (0, b, c, d) = b + c + d ≤ 3.
f (1, b, c, d) = 1 + b + c + d − bcd ≤ max { f (1, 0, c, d), f (1, 1, c, d)} .
Ta lại có
f (1, 0, c, d) = 1 + c + d ≤ 3


f (1, 1, c, d) = 2 + c + d − cd = 3 − (1 − c)(1 − d) ≤ 3.
Từ các kết quả trên ta có điều phải chứng minh.
Bài tốn 2.4 (Tổng quát). Cho x1 , x2 , . . . , xn ∈ [0; 1]. Chứng minh rằng
x1 + x2 + · · · + xn − x1 .x2 . . . ..xn ≤ n − 1.
Bài toán 2.5. Cho a, b, c là ba số thực không âm có tổng bằng 3. Chứng minh rằng
3( a2 + b2 + c2 ) + 4abc ≥ 13.

2


Hội thảo Khoa học, Sầm Sơn 28-28/09/2019
Dấu đẳng thức xảy ra khi nào?
Cách giải. Đặt
f ( a, b, c) = 3( a2 + b2 + c2 ) + 4abc − 13 = 14 − 6( ab + bc + ca) + 4abc.
Cách 1. Đặt

(3 − a )2
= t0
4
⇒ f ( a, b, c) = 14 − 6( ab + bc + ca) + 4abc = (4a − 6)t + 14 − 6a(3 − a) = g(t).
0 ≤ t = bc ≤

Ta có

1
( a + 3 − a )2
= >0
g(0) = 14 − 6a(3 − a) ≥ 14 − 6.
4

2
3
2
2a − 3a + 1
1
g ( t0 ) =
= ( a − 1)2 )( a + ) ≥ 0.
2
2

Dấu đẳng thức xảy ra khi a = 1 ⇒ b = c = 1
Vậy f ( a, b, c) ≥ 0. Dấu đẳng thức xảy ra khi a = b = c = 1.
Cách 2. Ta có

( a + b − c)(b + c − a)(c + a − b) ≤ abc ⇒ (3 − 2a)(3 − 2b)(3 − 2c) ≤ abc
4
⇒ 12( ab + bc + ca) − 8abc − 27 ≤ abc ⇒ 4abc ≥ [4( ab + bc + ca) − 9]
3
6 − 2( ab + bc + ca)
6−6
⇒ f ( a, b, c) ≥

= 0.
3
3
Dấu đẳng thức xảy ra khi a = b = c = 1.
Bài toán 2.6. Cho x,y, z, t thay đổi thuộc đoạn [0; 1]. Chứng minh rằng
x2 y + y2 z + z2 t + t2 x − ( xy2 + yz2 + zt2 + tx2 ) ≤

8

.
27

Cách giải. Đặt
f ( x, y, z, t) = x2 (y − t) + y2 (z − x ) + z2 (t − y) + t2 ( x − z).
Vì (y − t) + (z − x ) + (t − y) + ( x − z) = 0, nên trong 4 số: (y − t), (z − x ), (t − y), ( x −
z) = 0 phải có một số khơng âm, chẳng hạn là y − t ≥ 0.
Nếu y − t = 0 thì f ( x, y, z, t) = 0, nên bất đẳng thức hiển nhiên đúng.
Nếu y > t thì f ( x, y, z, t) = x2 (y − t) + y2 (z − x ) + z2 (t − y) + t2 ( x − z), là hàm lồi
theo x trên [0; 1], nên ta có f ( x, y, z, t) ≤ max { f (0, y, z, t), f (1, y, z, t)} .
Ta có f (0, y, z, t) = z(y − t)(y + t − z).
Nếu y + t − z ≤ 0 thì bất đẳng thức hiển nhiên đúng
Nếu y + t − z > 0 thì theo AM- GM, ta có
8y3
8
( z + y − t + y + t − z )3
f (0, y, z, t) = z(y − t)(y + t − z) ≤
=
≤ .
27
27
27

3


Hội thảo Khoa học, Sầm Sơn 28-28/09/2019
Dấu bằng khi
2 1
0, 1, ,

3 3

( x, y, z, t) =

.

Ta lại có
f (1, y, z, t) = (1 − z)t2 + (z2 − 1)t + y2 z − y2 + y − yz2 .
Dễ thấy đây là hàm lồi
max { f (1, y, z, 1), f (1, y, z, 0)} .
Ta có

theo

t

trên

[0; 1], nên ta có



f (1, y, z, t)

f (1, y, z, 0) = yz2 + y2 z − y2 + y ≤ max { f (1, y, 0, 0), f (1, y, 1, 1)} <

8
.
27


f (1, y, z, 1) = z2 (1 − y) + z(y2 − 1) − y2 + y ≤ max { f (1, y, 0, 0), f (1, y, 1, 1)} <

8
.
27

Cách 2. f ( x, y, z, t) = x2 (y − t) + y2 (z − x ) + z2 (t − y) + t2 ( x − z) = (y − t)( x − z)( x +
z − y − t ).
- Nếu cả ba nhân tử không dương bất đẳng thức hiển nhiên đúng.
- Nếu có một nhân tử khơng dương và hai nhân tử khơng âm thì bất đẳng thức hiển
nhiên đúng.
- Nếu cả ba nhân tử khơng âm thì theo AM - GM suy ra điều phải chứng minh.
- Nếu có hai nhân tử âm và một nhân tử dương thì đổi dấu hai nhân tử đó ta lại qui
về trường hợp cả ba nhân tử đều khơng âm.
Bài tốn 2.7. Cho ba số không âm a, b, c thoả mãn a + b + c = 3. Chứng minh rằng
a2 + b2 + c2 + abc ≥ 4.
Cách giải. f (t) = (c − 2)t + 2c2 − 6c + 5, 0 ≤ ab = t ≤

( c − 3)2
.
4

3 2 1
1
2
Ta có f (0) = 2(c − ) + > 0 và f (t) = (c − 1) (c + 2) ≥ 0.
2
2
4
Bài toán 2.8. Cho x, y, z là các số thực dương sao cho x + y + z = 1. Chứng minh rằng

1
1
1
27
+
+
≤ .
1 − xy 1 − yz 1 − zx
8

(1)

(1) ⇔ F = 11( xy + yz + zx ) − 19xyz + 27( xyz)2 − 3 ≤ 0.

(2)

Cách giải.
GT

⇒ xyz ≤

1
(1) ⇒ F ≤ 11( xy + yz + zx ) − 19xyz + ( xyz) − 3 = 11( xy + yz + zx ) − 18xyz − 3.
27
(3)
F ≤ f (t) = (11 − 18x )t + 11x (1 − x ) − 3, 0 ≤ t = yz ≤

4

(1 − x )2

= t0 .
4


Hội thảo Khoa học, Sầm Sơn 28-28/09/2019
Ta có f (0) = 11x (1 − x ) − 3 ≤ −
f (t0 ) = 11x (1 − x ) +

1
< 0.
4

−18x3 + 3x2 + 4x − 1
( x − 1/3)2 (18x + 9)
(1 − x )2
(11 − 18x ) − 3 =
=−
≤ 0.
4
4
4

Cách 2. (Phương pháp đồng bậc). (3) ≤ 0 ⇔ 11( x + y + z)( xy + yz + zx ) − 18xyz −
3( x + y + z )3 ≤ 0

⇔ K = 3( x3 + y3 + z3 ) + 3xyz − [2( x2 y + x2 z + y2 x + y2 z + z2 x + z2 y)] ≥ 0.

(4)

Mà x3 + y3 + z3 ≥ 3xyz ⇒ K ≥ 2[ x3 + y3 + z3 + 3xyz − ( x2 y + x2 z + y2 x + y2 z +

z2 x + z2 y)] ≥ 0. Đúng theo theo bất đẳng thức Schur.
Bài toán 2.9. Chứng minh rằng 5( a2 + b2 + c2 ) ≤ 6( a3 + b3 + c3 ) + 1, ở đó a, b, c là các
số thực không âm sao cho a + b + c = 1.
Cách giải. Viết lại bất đẳng thức đã cho dưới dạng bc(9a − 4) + 4a2 − 4a + 1 ≥ 0.
(1 − a )2
2
Đặt f (t) = (9a − 4)t + 4a − 4a + 1, t = bc, 0 ≤ t ≤
= t0 .
4
Ta có

2

f (0) = (2a − 1) ≥ 0
2
3
2
 f (t0 ) = 9a − 6a + a = a(3a − 1) ≥ 0.
4
4
Cách 2. (Đồng bậc + Schur)

⇔ a3 + b3 + c3 + 3abc ≥ a2 (b + c) + b2 (c + a) + c2 ( a + b).
Bài toán 2.10 (USAMO - 79). Với x, y, z là các số thực không âm thoả mãn x + y + z = 1.
Chứng minh rằng
x3 + y3 + z3 + 6xyz ≥ 1/4.

Cách giải. Viết lại bất đẳng thức đã cho dưới dạng −12(1 − 3x )yz + 12x2 − 12x + 3 ≥ 0.
(1 − x )2
Đặt: 0 ≤ t = yz ≤

= t0 và f (t) = −12(1 − 3x )t + 12x2 − 12x + 3. Ta có
4
2

f (0) = 3( x − 1/2) ≥ 0
f (t0 ) = 9x3 − 9x2 + 3x = 3x ( x − 1)2 ≥ 0.
Suy ra điều phải chứng minh.
Cách 2. (Đồng bậc + Schur) ⇔ x3 + y3 + z3 + 3xyz + 3xyz ≥ x2 (y + z) + y2 (z + x ) +
2
z ( x + y ).
Bài toán 2.11. Chứng minh rằng nếu x, y, z là các số thực không âm sao cho x + y + z = 1,
thì 4( x3 + y3 + z3 ) + 15xyz ≥ 1. Dấu đẳng thức xảy ra khi nào?
Cách giải. Viết lại bất đẳng thức đã cho dưới dạng
yz

27
1
3
x − 3 + (1 − x ) + x3 − ≥ 0.
4
4

5


Hội thảo Khoa học, Sầm Sơn 28-28/09/2019
Đặt
0 ≤ t = yz ≤

f (t) =

Ta có







(1 − x )2
= t0
4

27
1
x − 3 t + (1 − x )3 + x 3 − .
4
4
3
(2x − 1)2 ≥ 0
4
3x (3x − 1)2
f ( t0 ) =
≥ 0.
16
f (0) =

Dấu đẳng thức xảy ra khi ( x, y, z) = (1/3, 1/3, 1/3) hoặc tất cả các hoán vị của bộ ba
( x, y, z) = (0, 1/2, 1/2).
Bài toán 2.12 (BMO - 79). Với a, b, c ≥ 0 thoả mãn a + b + c = 1. Chứng minh rằng
7( ab + bc + ca) ≤ 2 + 9abc.

Cách giải. Viết lại bất đẳng thức đã cho dưới dạng 2 − 7x (1 − x ) + yz(9x − 7) ≥ 0. Đặt
(1 − x )2
f (t) = (9x − 7)t + 2 − 7x (1 − x ), với 0 ≤ t = yz =
= t0 . Ta có
4

f (0) = 7x2 − 7x + 2 ≥ 1/4 > 0

2
 f (t0 ) = 1 (9x3 + 3x2 − 5x + 1) = ( x + 1)(3x − 1) ≥ 0.
4
4
Cách 2. (Đồng bậc + Schur) ⇔ 2( x3 + y3 + z3 ) ≥ ∑ ( xy( x + y). Mà 2( x3 + y3 + z3 ) ≥
cyclic

x3

+ y3

+ z3

+ 3xyz ≥ ∑ ( xy( x + y), (đúng theo Schur).
cyclic

Bài toán 2.13. Cho x, y, z ∈ [0; 2] sao cho x + y + z = 3. Chứng minh rằng
x2 + y2 + z2 ≤ 5.
Cách giải. Cách 1. Giả sử x = max { x, y, z} ⇒ 1 ≤ x ≤ 2

⇒ x2 + y2 + z2 ≤ x2 + y2 + z2 + 2yz = x2 + (y + z)2 = x2 + (3 − x )2 = 2( x − 1)( x − 2) + 5 ≤ 5.
Cách 2. Giả sử x = max { x, y, z} ⇒ 1 ≤ x ≤ 2

2
2
Suy ra: x2 + y2 + z2 = x2 + (3 − x ) − 2yz = x2 + (3 − x ) − 2t = f (t), 0 ≤ t = yz ≤
(3 − x )2
= t0 .
4

 f (0) = x2 + (3 − x )2 ≤ 5, 1 ≤ x ≤ 2
Ta có
Suy ra điều phải chứng minh.
(3 − x )2
9

f ( t0 ) = x 2 +
≤ .
2
2
a+b+c = 0
Cách 3. Đặt a = x − 1, b = y − 1, c = z − 1 ⇒
| a| , |b| , |c| ≤ 1.
Ta có x2 + y2 + z2 = a2 + b2 + c2 + 3 ≤ | a| + |b| + |c| + 3. Vì a + b + c = 0 suy ra trong
3 số a, b, c có hai số cùng không âm hoặc cùng không dương, chẳng hạn là hai số a, b.

6


Hội thảo Khoa học, Sầm Sơn 28-28/09/2019
Suy ra | a| + |b| = | a + b| ⇒ | a| + |b| + |c| = | a + b| + |c| = 2 |c| ≤ 2 ⇒ | a| + |b| +
|c| + 3 ≤ 2 + 3 = 5. Đpcm.
Cách 4. Ta có

(2 − x )(2 − y)(2 − z) ≥ 0 ⇒ 2( xy + yz + zx ) − xyz − 4 ≥ 0 ⇒ −2( xy + yz + zx ) ≤
−4 − xyz
⇒ x2 + y2 + z2 = ( x + y + z)2 − 2( xy + yz + zx ) ≤ 9 − 4 − xyz ≤ 5. Đpcm.
Bài toán 2.14. Cho các số dương x, y, z thoả mãn x + y + z = 1. Chứng minh rằng
9xyz + 1 ≥ 4( xy + yz + zx ).
Cách giải. Viết lại bất đẳng thức đã cho dưới dạng yz(9x − 4) + −4x (1 − x ) ≥ 0.
y+z 2
1−x 2
Đặt 0 ≤ t = yz ≤
=
= t0 và f (t) = (9x − 4)t + 1 − 4x (1 − x ).
2
2
Ta có sẽ chứng tỏ rằng f (0) ≥ 0 và f (t0 ) ≥ 0.
x (3x − 1)2
2
Thật vậy f (0) = 1 − 4x (1 − x ) = (2x − 1) ≥ 0 và f (t0 ) =
≥ 0. Suy ra
4
đpcm.
Bài toán 2.15 (Romanian IMO slecton, test - 75). Tìm giá trị lớn nhất của biểu thức
1
a1 (1 − a2 ) + a2 (1 − a3 ) + · · · + an (1 − a1 ), với ≤ ai ≤ 1, i = 1, 2, . . . , n.
2
Cách giải. Đặt f ( a1 , a2 , . . . , an ) = a1 (1 − a2 ) + a2 (1 − a3 ) + · · · + an (1 − a1 ).
1
Suy ra f ( a1 , . . . , an ) ≤ max f ( , a2 , . . . , an ), f (1, a2 , . . . , an ) .
2
1
1

1
Ta có f ( , a2 , . . . , an ) = (1 − a2 ) + a2 (1 − a3 ) + · · · + an và f (1, a2 , . . . , an ) = (1 −
2
2
2
a 2 ) + a 2 (1 − a 3 ) + · · · + a n −1 (1 − a n )
a2
an
1
1
1
+

≥ 0 ⇒ f ( , a2 , . . . , a n ) ≥
⇒ f ( , a2 , . . . , an ) − f (1, an , . . . , an ) =
2
2
2
2
2
f (1, an , . . . , an ).
1
Suy ra f ( a1 , a2 , . . . , an ) ≤ f ( , a2 , . . . , an ).
2
1
Chứng minh tương tự, ta có f ( a1 , a2 , . . . , an ) ≤ f ( a1 , a2 , . . . , ai−1 , , ai , . . . , an ), (i =
2
2, . . . , n).
1 1
1

n
1
Vậy f ( a1 , a2 , . . . , an ) ≤ f ( , , . . . , ) = . Dấu đẳng thức xảy ra khi ai = .
2 2
2
4
2
n

1
1
Cách 2. Đặt ai = + ci , ở đó 0 ≤ ci ≤ , i = 1, 2, . . . , n. Thì f ( a1 , a2 , . . . , an ) =
2
2
n
n
1
1
n 1 n
1 n
+ ci
− ci+1 = + ∑ ci − ∑ ci+1 − ∑ ci ci+1 . Trong đó cn+1 = c1 .

2
4 2 i =1
2 i =1
i =1 2
i =1
n
n

n
Suy ra f ( a1 , a2 , . . . , an ) = − ∑ ci ci+1 ≤ . Dấu đẳng thức xảy ra khi ci = 0, i =
4 i =1
4
1, 2, . . . , n.
Cách 3. f ( a1 , a2 , . . . , an ) = a1 (1 − a2 ) + a2 (1 − a3 ) + · · · + an (1 − a1 ) = a1 + a2 + · · · +
a n − a1 a2 − a2 a3 − · · · − a n a1 .
Ta có
1
1
− ai
− ai+1 ≥ 0, (i = 1, 2, . . . , n, an+1 = a1 ).
2
2

7


Hội thảo Khoa học, Sầm Sơn 28-28/09/2019
Suy ra
1
1
1
n
1 n
≥ a i + a i +1 − a i a i +1 ⇒ ≥ ∑ ( a i + a i +1 ) −
4
2
2
4

2 i=

n

∑ a i a i +1

n

=

i =1

n

∑ ai −

∑ a i a i +1

i =1

.

i =1

Bài toán 2.16. Cho ai ∈ [−2008; 2008]. Tìm min của
S = a1 a2 + a2a3 + · · · + a29 a1 .

Cách giải. Đặt f ( a1 , a2 , . . . , a29 ) = a1 a2 + a2 a3 + · · · + a29 a1 .
Với mỗi bộ
( a1 , . . . , ai−1 , ai+1 , . . . , a29 ) ∈ [−2008; 2008]28 ,

cố định ta coi f ( a1 , a2 , . . . , ai , . . . , a29 ) như là hàm tuyến tính của ai , (i = 1, . . . , 29). Vậy
f ( a1 , a2 , . . . , a29 )
đạt giá tri nhỏ nhất tại ai ∈ {−2008; 2008} .
Xét 29 tích ai ai+1 với ai ∈ {−2008; 2008} , ( a30 = a1 ). Suy ra tồn tại một tích nhận
giá trị dương. Giả sử là
a1 a2 > 0

⇒ a1 a2 = 20082 ⇒ min f ( a1 , a2 , . . . , a29 ) ≥ 20082 + a2 a3 + · · · + a29 a1
≥ 20082 − 20082 − · · · − 20082 = −27 · 20082 .
28

Với a1 = a2 = 2008, a3 = −2008, a4 = 2008, . . . , a28 = 2008, a29 = −2009, thì dấu bằng
xảy ra.
Vậy min S = −27 · 20082 .
Bài toán 2.17. Cho ba số thực dương a, b, c thoả mãn a + b + c = 1, chứng minh rằng
7
1
1
1

+
+
.
1 + abc
a + bc b + ca c + ab

Cách giải. a + bc = a( a + b + c) + bc = ( a + b)( a + c) = (1 − b)(1 − c). ⇒ bđt ⇔
7( ab + bc + ca) ≤ 2 + 9abc.
Bài toán 2.18 (USAMO - 77). Cho a, b, c, d, e ∈ [ p, q,] p,q0. Chứng minh rằng


( a + b + c + d + e)

1 1 1 1 1
+ + + +
a b
c
d
e

≤ 25 + 6

Cách giải.
F = ( a + b + c + d + e)

8

1 1 1 1 1
+ + + +
a b
c
d
e

p

q

q
p


2

.


Hội thảo Khoa học, Sầm Sơn 28-28/09/2019

= 25 +
e
a
+ −2 +
e
a

+

a b
a c
+ −2 +
+ −2 +
b a
c
a

b c
+ −2 +
c b

b d
+ −2 +

d b

a d
+ −2
d
a

b
e
+ −2 +
e
b

c
c e
d
+ −2 +
+ −2 +
d
c
e c

Đặt
f ( x, y) =

x y
+ − 2 , x, y ∈ [ p, q].
y x

Ta có

f x ( x, y) > 0, f y ( x, y) > 0 ⇒ f ( x, y)
lồi theo từng biến

⇒ f ( x, y) ≤ max { f ( p, q), f (q, p), f ( p, p), f (q, q)} =
, vì
f ( p, p) = f (q, q) = 0, f ( p, q) = f (q, p) =

( p − q )2
pq

( p − q )2
.
pq

Suy ra max F đạt tại a, b, c, d, e ∈ { p, q} và trong số C52 = 10 cặp biến, ta có số cách
chọn ra cặp biến có hai biến khác nhau nhiều nhất là 6 cặp.
p
q 2
1 1 1 1 1
+ + + +
≤ 25 + 6

Vậy ( a + b + c + d + e)
.
a b
c
d
e
q
p

Cách 2. Ta sẽ chứng minh với các số dương x ≤ y ≤ z, ta có
f ( x, y) + f (y, z) ≤ f ( x, z) ⇔

x y
+ −2 +
y x

y z
+ −2
z y



x
z
+ − 2.
z
x

x y
y
x
x
y
y
z
z
x
≤ 0, vì , ≤ 1. Tương tự + − −
+ − −1 =

−1 1−
y
z
z
y
z
y z
x
y
x
y
z
1=
−1 1−
≤ 0, suy ra đpcm.
x
y
Sử dụng kết quả trên ta được (giả sử p ≤ a ≤ b ≤ c ≤ d ≤ e ≤ q ):
Để ý:

f ( p, a) + f ( a, b) + f (b, c) + f (c, d) + f (d, e) + f (e, q) ≤ f ( p, q)
f ( p, a) + f ( a, c) + f (c, e) + f (e, q) ≤ f ( p, q)
f ( p, a) + f ( a, d) + f (d, q) ≤ f ( p, q)
f ( p, a) + f ( a, e) + f (e, q) ≤ f ( p, q)
f ( p, b) + f (b, d) + f (d, q) ≤ f ( p, q)
f ( p, b) + f (b, e) + f (e, q) ≤ f ( p, q)
Cộng các bất đẳng thức này ta có điều phải chứng minh (vì f ( x, y) ≥ 0; x, y ≥ 0 ).
Tổng quát: Cho p, q là hai số dương với p < q. Chứng minh rằng nếu xi ∈ [ p; q] (i =
1, 2, . . . , n) là n số (n ≥ 2), thì ta có:
1

1
q
p 2
1
+
+···+
≤ n2 + K n

, trong đó Kn =
( x1 + x2 + · · · + x n )
x1
x2
xn
p
q

n2


, n = 2m
4
2

 n − 1 , n = 2m + 1.
4

9

d
+

e


Hội thảo Khoa học, Sầm Sơn 28-28/09/2019

3

Bài toán tương tự

Bài 3.1. Cho a, b, c A, B, C là các số thực không âm sao cho a + A = b + B = c + C = k.
Chứng minh rằng aB + bC + cA ≤ k2 .
Cách giải. Bđt ⇔ a(k − b) + b(k − c) + c(k − a) ≤ k2 . Với chú ý ( a, b, c) ∈ [0; k ]3 .
Bài 3.2 (Bulgarian Mathematical Olympiad - 95). Cho số tự nhiên n ≥ 2 và 0 ≤ xi ≤
1, ∀i = 1, 2, . . . , n.
n
Chứng minh rằng ( x1 + x2 + · · · + xn ) − ( x1 x2 + x2 x3 + · · · + xn x1 ) ≤
; ( [ x ] là
2
phần nguyên của x).
Bài 3.3 (USAMO -80). Với a, b, c ∈ [0; 1]. Chứng minh rằng
b
c
a
+
+
+ (1 − a)(1 − b)(1 − c) ≤ 1.
b+c+1 c+a+1 a+b+1
α
là hàm lồi với x ≥ 0.
x+β

Tổng quát: Với ai ∈ [0; ], (i = 1, n). Chứng minh rằng

Cách giải. Với mọi α, β không âm, hàm số f ( x ) =

a1
an
+···+
+ (1 − a1 )(1 − a2 ) . . . (1 − an ) ≤ 1.
a2 + · · · + a n + 1
a 1 + · · · + a n −1 + 1

4

Kết luận

Đứng trước một bài toán việc định hướng phương pháp tiếp cận lời giải là rất cần
thiết. Phương pháp trình bày ở trên có thể vận dụng để giải một lớp bài toán về chứng
minh bất đẳng thức có điều kiện, mà các hàm số trong bất đẳng thức là các hàm lồi, hàm
lõm theo biến nào đó, có thể là biến qua ẩn phụ. Ở đây hàm số f ( x ) = ax + b là trường
hợp đặc biệt của hàm lồi, hàm lõm.

Tài liệu
[1] Titu Andreescu, Răzvan Gelca. Mathematical Olympiad Challenges.
[2] Mathematcal Rflections.
[3] Titu Andreescu, Vasile Cirtoaje, Gabriel Dospinescu, Mircea Lascu, Old and New
Inequalities, Gil Publishing House, 2004.

10




×